3 shapes are combined to form a composite figure. a semicircle is on the bottom, and then a small rectangle connects the semicircle to a triangle. how can you decompose the composite figure to determine its area? as three triangles and a circle as two triangles, a rectangle, and a circle as a triangle, a pentagon, and a semicircle as a triangle, a rectangle, and a semicircle

Answers

Answer 1

Answer:

  (d)  as a triangle, a rectangle, and a semicircle

Step-by-step explanation:

The description of the shape tells you how to decompose it. The shape is described as a semicircle, a rectangle, and a triangle. The appropriate decomposition is ...

  as a triangle, a rectangle, and a semicircle

Answer 2

Answer:

d

Step-by-step explanation:


Related Questions

helpppp :) 50 points
An equation is shown below:

4(2x – 5) = 4

Part A: How many solutions does this equation have? (4 points)

Part B: What are the solutions to this equation? Show your work. (6 points)

Answers

Answer:

x = 3

Step-by-step explanation:

A only 1 solution

B

4(2x - 5) = 4 ( divide both sides by 4 )

2x - 5 = 1 ( add 5 to both sides )

2x = 6 ( divide both sides by 2 )

x = 3

Answer:

[tex]\boxed{\sf{x=3}}[/tex]

Step-by-step explanation:

Isolate it by the term of x from one side of the equation.

Part A. There will be one solution.Part B.

4(2x-5)=4

First, divide by 4 from both sides.

[tex]\sf{\dfrac{4\left(2x-5\right)}{4}=\dfrac{4}{4}}[/tex]

Then, solve.

4/4=1

Rewrite the problem down.

2x-5=1

Add by 5 from both sides.

[tex]\sf{2x-5+5=1+5}[/tex]

Solve.

1+5=6

2x=6

Divide by 2 from both sides.

2x/2=6/2

Solve.

Divide the numbers from left to right.

6/2=3

x=3

Therefore, the correct answer is x=3.

I hope this helps you! Let me know if my answer is wrong or not.

Help is number 2 correct and help for number one​

Answers

Number 2 is incorrect :

[tex]\frac{-25x^{3}+15x }{5x} = \frac{-25x^{3}}{5x} + \frac{15x}{5x} = -5x^{2} + 3[/tex]

Number 1 :

[tex](-5)^{3} . 13^{-2} . (-5) .13^{9} = -125 . \frac{1}{13^{2}} .(-5).13^{9} = 625 . \frac{13^{9}}{13^{2}} = 625 . 13^{7}[/tex] (i dont think we can bare calculate this cause it;s too big ;-;)

PLEASE HELP ME! THERE IS A PICTURE

Answers

B. 1/40…. You divide 1/2 by 20 to find out how much hand sanitizer was used on each desk. You can keep change flip meaning keep the 1/2 change the division to multiplication and flip the 20 to 1/20 then multiply. 1/2 by 1/20 which equals 1/40.

Answer:

I think 10

letter C.

Step-by-step explanation:

divide *2

Pls help me
AND

PLS FOLLOW THE RULES

TROLL FOR POINTS= REPORT

FIRST AND CORRECT ANSWER= BRAINLIEST

( NO LINKS ) at all

Answers

Pythagorean Theorem: a^2 + b^2 = c^2

a = 23 m

b = 11 m

c^2 = (23 m)^2 + (11 m)^2 = 650 m^2

c = sqrt(650 m^2) = 25.5 (rounded to nearest tenth)

hello, me again polease help me with this, and show work

Answers

Answer:

Hope it helps u

Step-by-step explanation:

1.7

1.749

1.75

1.775

1.796

1.84

Answer:

the order would go 1.7, 1.749, 1.75, 1.775, 1.796, then 1.84.

Step-by-step explanation:

becuase if you ignore how long the numbers are, and focus on numerical order like 1, 2, 3, and apply that to this, this becomes the order.

Find the value of log 1 2 462 to four decimal places. –8.8517 –0.1130 0.1130 8.8517

Answers

The logarithm value of log 1/2 462 to four decimal places is  –8.8517.

What is a logarithm?

The logarithm is the mathematical inverse function to exponentiation.

The logarithm of a number x is the exponent to another fixed term.

It is given the log of 462 to a base of 1/2.

[tex]log_{0.5} 462 = \frac{log462}{log0.5}[/tex]

 = –8.8517

Therefore, the logarithm value of log 1/2 462 to four decimal places is  

–8.8517.

Learn more about logarithm :

https://brainly.com/question/3181916


Fifteen less than the square of a number is the same
as twice the number. Find the number.

Answers

Call the number : a

We have : [tex]a^{2} - 15 = 2a[/tex]
[tex]- > a^{2} - 2a = 15 - > a(a-2) = 15[/tex]

Now, we list all the numbers that can be multiplied to get 15 :

1 x 15 (and reverse)
3 x 5 (and reverse)

-1 x (-15) (reverse)
-3 x (-5) (a l s o r e v e r s e)

We substitute in :

If a = 1 -> a(a-2) = 1 x (-1) = -1 (not equal 15)
If a = 3 -> a(a-2) = 3 x 1 = 3 (not equal 15)
If a = -1 -> a(a-2) = -1 x (-3) = -3 (also not equal 15)
And if a = -3 -> a(a-2) = -3 x (-5) = 15 (satisfied)

So, the number is -3

Recheck : -3(squared) - 15 = 9 - 15 = -6
Twice of -3 = -6

Hello!

[tex]========================================[/tex]

First of all, let the unknown number be z.

Then, "the square of z" means "z squared" which is the same as "z times z" or just z².

Now, "fifteen less than the square of z" means we subtract 15 from z²:

z²-15

Then, this equals twice the number, or 2 times z:

z²-15=2z

[tex]========================================[/tex]

Notes:-Hope everything is clear.Let me know if you have any questions!Always remember: [tex]\boxed{Knowledge~is~power!}[/tex]Enjoy your day!Answered by[tex]\boxed{An~Emotional~Helper}[/tex]

:-)

Prism M is a dilation of Prism N. The height of Prism M is 4 3/4 ft, and the volume of Prism M is 68 2/5 ft³. The height of Prism N is 2 3/8 ft. What is the volume of Prism N?Enter your answer as a mixed number in simplest form by filling in the boxes.

Answers

Answer:

  8 11/20 ft³

Step-by-step explanation:

For dilated figures, the ratio of volumes is the cube of the ratio of corresponding linear dimensions.

  Vn/Vm = (Hn/Hm)³

  Vn = Vm(Hn/Hm)³ = (68 2/5 ft³)((2 3/8 ft)/(4 3/4 ft))³

  Vn = (342/5 ft³)((19/8)/(19/4))³ = (342/5 ft³)(1/2)³ = 342/40 ft³

  Vn = 8 11/20 ft³

The volume of Prism N is 8 11/20 ft³.

Answer:

The answer is 8 11/20.

Volt Electronics sells a Y-Box game console for $207. The store markup on thr Y-Box is 38%. What was the wholesale price that the store paid for it?​

Answers

The wholesale price that the store paid for the Y-Box game is $150.

What is the wholesale price?

When the price of an item is marked up, it means that the price of the item was increased by a certain percentage. The good then becomes more expensive.

Wholesale price = selling price / ( 1 + percentage mark up)

$207 / (1+ 38)

$207 / (1.38) = $150

To learn more about profit, please check: https://brainly.com/question/26181966


Given the equation of a circle. X^2+ y^2 - 6x + 2y +1 = 0, find the coordinates of the center and the length of the radius.
Coordinates for the center=
Length of the radius =

Answers

Answer:

I know the answer but you may select me as Brainliest

Answer:
Center: (3, -1)
Radius: 3

Steps:
Rewrite in standard form to find the center
(h,k) and radius.

x^2 + y^2 - 6x + 2y + 1 = 0
Standard Form: (x - 3)^2 + (y + 1)^2 = 9

Center: (3, -1)
Radius: 3

graph "-3n+4<25" on a line plot

Answers

Answer:The solution is in the attached file

Step-by-step explanation:

The height of the prism is the Blank between the bases.

Answers

Answer:

so how do i find the earea

Step-by-step explanation:

In the figure below, mR is 66°, and mT is 130°. Note: Figure is not drawn to scale. What is mQ? A. 24° B. 50° C. 64° D. 116°

Answers

Answer:

b

Step-by-step explanation:

i just think so correct me if im wrong

Tiana bought 3 video games that each cost
the same amount. The tax on each video
game was $1.29. She paid a total of $68.37
after tax. What was the price of each video
game, before tax?
a

Answers

well, we know she paid a total after tax of $68.37, we also know that each game had a tax of $1.29, so 3 games that'll be 3 * 1.29 = 3.87.

so if we take out the tax from 68.37, that'll be 68.37 - 3.87 = 65 bucks.

we know each game cost the same amount so 65 ÷ 3 ≈ 21.67 before tax.

Tiana bought 3 video games at a cost of $22.36 each, before tax. The total amount paid after tax was $68.37.

Given that,

Tiana bought 3 video games.

The cost of each video game was the same.

The tax on each video game was $1.29.

Tiana paid a total of $68.37 after tax.

To find the price of each video game before tax,

Start by subtracting the total tax from the total amount paid.

Tiana paid a total of $68.37 after tax and the tax on each video game was $1.29.

So, subtract the tax from the total amount to get the price before tax.

$68.37 - $1.29 = $67.08.

Since Tiana bought 3 video games,

Divide the total price before tax by 3 to find the price of each video game.

$67.08 ÷ 3 = $22.36.

Hence, the price of each video game before tax was $22.36.

To learn more about subtraction visit:

https://brainly.com/question/17301989

#SPJ3

Two rectangles share a common side as shown.
Determine whether each expression can be used to determine the
total area, in square feet, of the two combined rectangles.
ft
Select Yes or No for each expression.
7.52 + 20
o Yes
Ο Νο
*
7.5 ft
x ft
27.5 + 2
Yes
Ο Νο
20 (7.5 + x)
o Yes
Ο Νο
150 + 3
Yes
Ο Νο
202 + 150
O Yes
No

Answers

Answer:

yes they are

Step-by-step explanation:

Jill bought 3 pints of blueberries for $3.02 each. What is the best estimate of how much Jill spent

Answers

Answer: 9.06

Step-by-step explanation:

Do; 3.02+3.02+3.02

This adds up to 9.06

I am thinking of a number. The sum of its digits is divisible by 2. My number is a multiple of 11. My number is greater than 4 x 5 but less than 7 x 8+ 23. The number is a multiple of 3. What is my number? Is it possible to have more than one answer? ​

Answers

Answer:

66

Step-by-step explanation:

Number:xy

20<xy<79

Multiples of 2,3,11

The least common multiple of this number is 66.

Use your TI-83 to determine the necessary t* value for a 90% confidence interval
based on a sample size of 55 and a sample standard deviation of 7.42.

2.005
1.669
1.96
1.645
2.67

Answers

Answer:

Step-by-step explanation:

It is 1.645. My calculator is a little off though; see attached picture.

Pls help dont need an explanation just an answer

Answers

At y-intercept x=0.
Y-intercept = 1

(0,1). & (-1 ,-4)

Slope =y2-y1/(x2-c1)
=(-4-1)/(-1-0)
=-5/-1
Slope =5
= (

Answer:

[tex]\boxed{\text{Slope = 5}}[/tex]

[tex]\boxed{\text{y-intercept = 1}}[/tex]

Step-by-step explanation:

To find the slope of the line, we must pick any two points on the line. Using those two points, we will calculate the Rise and the Run. Then, we will use the slope formula (Rise/Run) to find the slope. The y-intercept is the intersection of the point on the y-axis.

[tex]\text{My chosen points: (0,1) and (1,6)}[/tex]

[tex]\text{Rise = 5; Run = 1}[/tex]

≡≡≡≡≡≡≡≡≡≡≡≡≡≡≡≡≡≡≡≡≡≡≡≡≡≡≡≡≡≡≡≡≡≡≡≡≡≡≡≡≡≡≡≡≡≡≡≡≡≡≡≡≡

[tex]\text{Slope}= \dfrac{\text{Rise}}{\text{Run}}[/tex]

➡ [tex]\bold{Slope = \frac{5}{1} = \underline{5}} \ \ \ \ \ \ \ \ \ \ \ \ \ \ \ \ \ \ \ \ \ \ \ \ \ \ \ \ \ \ \ \ \ \ \ \ \ \ \ \ \ \ \ \ \ \ \ \ \ \ [\text{Rise = 5; Run = 1}][/tex]

[tex]\underline{........................................................................................................................}[/tex]

[tex]\text{Intersection on y-axis:} \ (0,1)[/tex]

➡ [tex]\text{y-intercept} \rightarrow (0,\bold{ 1}) \rightarrow \bold{\underline{{1}}}}[/tex]

≡≡≡≡≡≡≡≡≡≡≡≡≡≡≡≡≡≡≡≡≡≡≡≡≡≡≡≡≡≡≡≡≡≡≡≡≡≡≡≡≡≡≡≡≡≡≡≡≡≡≡≡≡

What points should I circle and in which point do they intersect?

I WILL MARK BRAINLIEST AND GIVE THANKS

Answers

Answer:

[tex]\displaystyle [3, -2][/tex]

Explanation:

All you do is substitute three in for [tex]\displaystyle x[/tex]to get the y-coordinate of [tex]\displaystyle -2.[/tex]This means that these equations meet at the ordered pair of [tex]\displaystyle [3, -2].[/tex]

I am joyous to assist you at any time.

Factor: 35b + 15c -10

Answers

Answer:

5(7b+3c-2)

Step-by-step explanation:

Common factor of 5

35b/5= 7b
15c/5= 3c
-10/5= -2

Take the common factor and multiply it by the quotients
5(7b+3c-2)

The rule for the number of fish in a home aquarium is 1 gallon of water for each inch of fish length. Marta's aquarium holds 39 gallons of water and Hank's aquarium hold 51 gallons of water. The aquarium holds two types of fish, fish A and fish B. If Marta bought 3 of fish A and 2 of fish B, and Hank bought 3 of fish A and 4 of fish B, how long is fish A and how long is fish B?

Answers

Answer: Fish A: 9 inches ; Fish B: 6 inches

Step-by-step explanation:

This can be turned into a systems of equation, where x equals the length of fish A and y equals the length of fish B:

3x + 2y = 39 → -3x - 2y = -39

3x + 4y = 51 → 3x +4y = 51

Combine: 0 + 2y = 12 → y = 6 inches.

Plug back in: 3x + 2(6) = 39 → 3x = 27 → x = 9 inches.

can you pls help me i need it?

Answers

Answer:

$114.75

Step-by-step explanation:

You have to multiply the hours by the wage. Looking at the time that the worker was in during the morning, it was a total of 4 hours. Since the wage is $13.50/hr, we would multiply 13.50 by 4.

13.50 * 4 = 54.00

So, now we have to add together the total hours in the afternoon. If we count the time, we get 4 1/2 hours. So, now we multiply 13.50 by 4.5.

13.50 * 4.5 = 60.75

Now, to find the total pay for that day, we add both the morning and the afternoon pay together.

54.00 + 60.75 = 114.75

Therefore, the pay for this day is $114.75.

Use the dot plot to answer the question.

A dot plot that has 3 dots at 1, 4 dots at 2, 3 dots at 3, 4 dots at 4, 5 dots at 5, & 3 dots at 6.

What is the mean of the data set in the dot plot?

Enter your answer as a number rounded to the nearest tenth, like this: 42.5

Answers

The mean of the data set is 2.20

The first term of a geometric sequence is 4 and grows exponentially by a factor of 3. Murphy writes out the terms and says that the sum of the 4th and 5th terms is​ 1,296. Explain​ Murphy's error and correct it. (Fill in the blanks)
Murphy added terms
____
together. The correct sum of the 4th and 5th terms is ___
​(Simplify your​ answer.)

Answers

The geometric sequence can increase or decrease exponentially

The correct sum of the 4th and the 5th terms is 432

How to determine the correct sum?

The given parameters are:

Initial value (a) = 4

Growth factor (r) = 3

The nth term of a geometric sequence is calculated as:

[tex]T_n = ar^{n-1}[/tex]

This gives

[tex]T_n = 4 * 3^{n-1}[/tex]

For the 4th term; we have:

[tex]T_4 = 4 * 3^{4-1}[/tex]

[tex]T_4 = 108[/tex]

For the 5th term, we have:

[tex]T_5 = 4 * 3^{5-1}[/tex]

[tex]T_5 = 324[/tex]

Add the 4th and the 5th terms

Sum = 108 + 324

Sum = 432

Hence, the correct sum of the 4th and the 5th terms is 432

Read more about geometric sequence at:

https://brainly.com/question/24643676

A triangular pyramid has a base shaped like an equilateral triangle. The legs of the equilateral triangle are all 1 yard long, and the height of the equilateral triangle is 0.9 yards. The pyramid's slant height is 1 yard. What is its surface area?

Answers

Answer:

base: 1/4 * 12^2 * √3 = 36√3

each face: 1/2 * 12 * 8 = 48

Now finish it off

Step-by-step explanations:

What are the y-intercept and the horizontal asymptote of g(x) = 4x + 3?
(0, 3); y = 4
(0, 4) ; y = 3
(0, 5) ; y = 4
(0, 7) ; y = 3

Answers

Using exponential function concepts, it is found that that the y-intercept and the horizontal asymptote are given by, respectively:

(0, 3); y = 4

What is an exponential function?

It is modeled by:

[tex]y = ab^x + c[/tex].

In which:

a is the initial value.b is the rate of change.y = c is the horizontal asymptote.

In this problem, the equation is:

[tex]y = 4^x + 3[/tex].

Hence the horizontal asymptote is y = 3, and the y-intercept is y when x = 0, hence:

[tex](0,y) = (0, 4^0 + 3) = (0, 4)[/tex]

Which means that the first option is correct.

More can be learned about exponential function concepts at https://brainly.com/question/25537936

Answer:

B. (0, 4) ; y = 3

Step-by-step explanation:

I took the exam for algebra 2

Four eighths of kelly’s beads are red, 18 are white, and the rest are blue.
what fraction of kelly’s beads are blue?

Answers

4/8=1/2 so Kelly blue beads are also 4/8. Sorry if I’m wrong.

Question 1-5
Sarah is demonstraing the idea that polynomials are closed under addition. Her work and explanation are shown.
Select an equation and phrases to complete the statement.
Part A:
Add 3x4 + 6x - 7 to
Equation A 2x - (2x)2 + 4x-1
Equation B (2x)2 + 4.4x - 9
Equation C (-8x)-2 - (3x)3 +7
Part B:
This is closed because
of the sum are
and the sum is a polynomial.

Answers

This is closed under addition because the sum of the polynomial produces another polynomial.

What is a polynomial?

A polynomial is an expression that involves the addition, subtraction and multiplication of variables.

Polynomials will be closed under an operation if the operation produces another polynomial.

(3x⁴ + 5x - 7) + (2x - (2x)² + 4x - 1) + [(-8x)⁻² - (3x)³ + 7]

= 3x⁴ - 27x³ - 4x² + 9x - 1 + (-8x)⁻²

This is closed under addition because the sum of the polynomial produces another polynomial.

Find out more on polynomial at: https://brainly.com/question/2833285

In the figure below, triangle ABC undergoes a reflection and a translation to become triangle PQR. In triangle ABC, m

The angle measures of triangle PQR are: m

Answers

Answer:

Step-by-step explanation:

A corresponds to angle P, so angle P measues 70 degrees.

Similarly, angle Q measures 60 degrees and angle R measures 50 degrees.

Other Questions
Why is it important to review the job posting before you complete your resum? to get the address of the company so you can tailor your resum to the position so you can add jobs youve had like that one to find contact information for your resum Western and central europe: 740,000 adults and children living with h i v and 12,000 deaths due to h i v. eastern europe and central asia: 1.7 million with h i v and 84,000 deaths. east asia: 750,000 with h i v and 43,000 deaths. south and southeast asia: 7.8 million with h i v and 90,000 deaths. oceania (including australia and new zealand): 81,000 with h i v and 4,000 deaths. middle east and north africa: 460,000 with h i v and 36,000 deaths. sub-saharan africa: 24.7 million with h i v and 2.1 million deaths. which area of the world has the most people living with hiv/aids? the middle east and north africa sub-saharan africa south and southeast asia eastern europe and central asia Describe the process used to balance an equation? Will give brainliest Given \tan A=\frac{11}{60}tanA= 6011 and that angle AA is in Quadrant I, find the exact value of \cos AcosA in simplest radical form using a rational denominator. Write -4/5 as a decimal number Which measurement is more precise? 45 fluid ounces or 79 cups How do scientists test theories Nosotros ________ deportes todos los das.A. jugamosB. jueganC. juegamosD. juegas Consider the enlargement of the complex figure. 2 complex figures. a corresponds to 18 centimeters, b corresponds to 24 centimeters, and a side with length 14 meters corresponds to a side with length 42 feet. what can be concluded about the scale factor and missing measures? check all that apply. the scale factor is 3. the scale factor is one-third. add the scale factor to 18 and 24 to find a and bs lengths. subtract the scale factor from 18 and 24 to find a and bs lengths. multiply 18 and the scale factor to find as length. divide 24 by the scale factor to find bs length. Write an equation in point-slope form of the line that passes through the point (5,6) with slope 3/5 How many years does a welder do in college? What is the solution to the inequality?12 < 8 + xO A. x < 20O B. x>4O c. x> 20O D. x A rule for creating a pattern is given below. The rule begins with a number called the input and creates a number called the output.Rule: Multiply the input by 5. Then subtract 4 from the result to get the output.Which input and output table works for the rule?Choose 1 answer:(Choice A)Input: 5 Output: 5(Choice B)Input: 3 Output: 7(Choice C)Input: 2 Output: 6 Choose the student with the correct answer. Pls Help ;-; A car initially travels at 10 m/s when passing the "acceleration line". The car then accelerates Q. at 8m/s2 until reaching a final velocity of 40 m/s. What is the car's displacement during the acceleration? answer choices 3.75 m 93.75 m 100 m 1590 m United States farms produced 2,460,000,000 bushels of soybeansin 1994. How many quarts is this? (A bushel is 32 quarts.)Pls u need this now If B = -6What is b - 5b - 7? Why is adole scence called a transition period? please me A regular deck of cards has 52 cards. assuming that you do not replace the card you had drawn before the next draw, what is the probability of drawing three aces in a row?